What is the value of the expression? 30 - 5 x 4 + 2

Answers

Answer 1

Answer:12

Step-by-step explanation:

5x4=20+2=22

30-22=12

I hope this helped, please mark brainliest

Answer 2

Answer: 12

Work: You need to use the PEMDAS Rule. The first thing you do is 5 x 4. After doing that, your new equation is 30 - 20 + 2. After doing that equation, you get 12.

I hope this helps, and Happy Holidays! :)


Related Questions

what is the rate of change?​

Answers


6
_
80

So for every 6 years you would get $80 in your savings account.

Hope this helped

A proportional relationship between y and x. When x = 3, y=−4 .

What is the constant of proportionality, k?

Enter your answer as a simplified or improper fraction in the box.

k =

Answers

Answer:

k = 4/3

Step-by-step explanation:

Formula for k: y/x

If y = 4 and x = 3, then we use the formula (y/x) in order to find the constant of proportionality:

k = 4/3

Approximately how many strides does it take to complete a marathon?
Choose 1 answer:

Answers

Male steps: 55,375
Female steps:62,926

Answer:

Step-by-step explanation:

determine the measure of each segment then indicate whether the statements are true or false

Answers

Answer:

[tex]d_{AB}\ne d_{JK}[/tex]

[tex]d_{AB}\ne \:d_{GH}[/tex]

[tex]d_{GH}\ne \:d_{JK}[/tex]

Therefore,

Option (A) is false

Option (B) is false

Option (C) is false

Step-by-step explanation:

Considering the graph

Given the vertices of the segment AB

A(-4, 4)B(2, 5)

Finding the length of AB using the formula

[tex]d_{AB}\:=\:\sqrt{\left(x_2-x_1\right)^2+\left(y_2-y_1\right)^2}[/tex]

        [tex]=\sqrt{\left(2-\left(-4\right)\right)^2+\left(5-4\right)^2}[/tex]

         [tex]=\sqrt{\left(2+4\right)^2+\left(5-4\right)^2}[/tex]

         [tex]=\sqrt{6^2+1}[/tex]

         [tex]=\sqrt{36+1}[/tex]

        [tex]=\sqrt{37}[/tex]

[tex]d_{AB}\:=\sqrt{37}[/tex]

[tex]d_{AB}=6.08[/tex] units        

Given the vertices of the segment JK

J(2, 2)K(7, 2)

From the graph, it is clear that the length of JK = 5 units

so

[tex]d_{JK}=5[/tex] units

Given the vertices of the segment GH

G(-5, -2)H(-2, -2)

Finding the length of GH using the formula

[tex]d_{GH}\:=\:\sqrt{\left(x_2-x_1\right)^2+\left(y_2-y_1\right)^2}[/tex]

         [tex]=\sqrt{\left(-2-\left(-5\right)\right)^2+\left(-2-\left(-2\right)\right)^2}[/tex]

          [tex]=\sqrt{\left(5-2\right)^2+\left(2-2\right)^2}[/tex]

          [tex]=\sqrt{3^2+0}[/tex]

           [tex]=\sqrt{3^2}[/tex]

[tex]\mathrm{Apply\:radical\:rule\:}\sqrt[n]{a^n}=a,\:\quad \mathrm{\:assuming\:}a\ge 0[/tex]

[tex]d_{GH}\:=\:3[/tex] units

Thus, from the calculations, it is clear that:

[tex]d_{AB}=6.08[/tex]  

[tex]d_{JK}=5[/tex]

[tex]d_{GH}\:=\:3[/tex]

Thus,

[tex]d_{AB}\ne d_{JK}[/tex]

[tex]d_{AB}\ne \:d_{GH}[/tex]

[tex]d_{GH}\ne \:d_{JK}[/tex]

Therefore,

Option (A) is false

Option (B) is false

Option (C) is false

which graph represents a function? please help

Answers

Answer:

C

Step-by-step explanation:

Answer:D

Step-by-step explanation: a function has only one output value for each input value

Please answer this question

Answers

Answer:

number: 2

Step-by-step explanation:

The answer should be 1/3 because when you get to the answer it’d give you 3/6. Those are divisible therefor you divide them by 3 and it gives you 1/3

Hope this helped and merry Christmas!

HELP ASAP! WILL MARK BRAINLIEST! PLEASE ANSWER CORRECTLY THIS IS VERY IMPORTANT!!!!

Answers

Answer:

You should put "no solution" into the parallel lines box,

you should put "infinite solutions" into the coinciding lines box,

you should put the "(0,0)" and the "(3,-2)" into the intersecting lines box.

Step-by-step explanation:

Susan went to dinner at San Felipe and ordered the ACP for $10.99. If she paid 8% tax, what was her total cost?

Please help!

Answers

Answer:

11.88 $ (approximately)

Step-by-step explanation:

10.99 is approximately 11

So

100$ …8$

1$ .… 8/100 $

11$… 8/100×11 $

22/25 $

0.88 $

So she total has to pay

11 + 0.88 $

11.88 $ (approximately)

Over a one week period in the summer, the price of a gallon of gas increased from $2.35 to $2.82. By what percent did the price of gas increase over that week?

Answers

The price of gas increases from $2.35 to $2.82 or by $0.47 ($2.82 - $2.35 = $0.47).
The increase as a percentage of the original price is given by
($0.47/$2.35) x 100% = 20%

The answer is 20%
Hope this helps :)

The percentage increase from $2.35 to $2.82 over the week in summer is 20%

Given Data

Initial Value = $2.35

Final Value = $2.82

We know that the expression for Percentage increase is given as

% Increase = Increment/Initial Value *100

% Increase = 2.82-2.35/2/35 * 100

% Increase =0.47/2/2.35 * 100

% Increase =0.2* 100

% Increase = 20%

Hence the percentage increase is 20%

Learn more about percent increase here:

https://brainly.com/question/11360390

Graph the line that has a slope of -9 and includes the point (1, -1).

Answers

Answer:

Step-by-step explanation:  you didnt put a Graph in your question

PLEASE HELP ASAP!!

j^4-5j^3+10j

Answers

Answer:

B) j(j^3-5j^2+10)

Step-by-step explanation:

Given: [tex]j^4-5j^3+10j[/tex]

Factor out j: [tex]j(j^3-5j^2+10)[/tex]

We cannot factor out anymore, so the final answer is B) j(j^3-5j^2+10)

If y=32 when x=8, find y when x=30.

Answers

Answer: 120

*How to solve*

Step 1: Find how many times larger 30 is than 8

30/8= 3.75

Step 2: Multiply by y

32x3.75=120


Hope this helps comment below for more questions:)

Find the missing side length
Round to the nearest tenth
The answer is 4.8
Please show all steps thank u

Answers

Answer:

x ≈ 4.8

General Formulas and Concepts:

Pre-Algebra

Order of Operations: BPEMDAS

Brackets Parenthesis Exponents Multiplication Division Addition Subtraction Left to Right

Equality Properties

Trigonometry

[Right Triangles Only] SOHCAHTOA[Right Triangles Only] cos∅ = adjacent over hypotenuse

Step-by-step explanation:

Step 1: Identify Variables

Angle measure = 58°

Adjacent side of angle = x

Hypotenuse = 9

Step 2: Solve for x

Substitute [cosine]:                    cos58° = x/9Isolate x:                                     9cos58° = xEvaluate:                                     4.76927 = xRewrite:                                      x = 4.76927Round:                                        x ≈ 4.8

Answer:

if you have the answer then wha.

Step-by-step explanation:

HELPP PLEASEEEEE 42 POINTS !!! what is 6( -3w + 1/3) =

Answers

Answer:

2 - 18w or -18w + 2

Step-by-step explanation:

6(-3w + 1/3)

Multiply using distrubutive property

-18w + 6(1/3)

Multiply 6 * 1/3 to get 6/3 or 2

-18w + 2

This is optional, but put the variable term on the right

2 + (-18w)

2 - 18w


Identify the angle pair from the diagram above: <1 and <5
Linear Pair

Answers

they are corresponding angles

You are renting a cabin that changes you a processing fee along with a daily rate. The total charge for this cabin is represented by the equation y=4x+35,where y represents the total cot and x represents the length of stay in days. How much would you be charged for 20 days at this cabin

Answers

Answer:

y= 115

Step-by-step explanation:

Step one:'

given data

The total charge for this cabin is represented by the equation

[tex]y=4x+35[/tex]

we are told that  y represents the total cost and x represents the length of stay in days

Required:

the value of y when x= 20

Step two:

substitute

[tex]y=4(20)+35\\\\y=80+35\\\\y=115[/tex]

please answer if u know

Answers

Answer:

9 dollars

Step-by-step explanation:

let x be per mile

the equation for solving this would be 0.5x + 3(flat fee)=amount you have to pay

The x is 12 since you have to go 12 miles

0.5(12)+3=amount you have to pay

6+3=amount you have to pay

$9=mount you have to pay

Can someone help me with sub multiple angles trigonometry pls​

Answers

Answer:

These photos have the formulas of sub multiple angle trigonometry.

Hope it helps:)

find the values of x and y

Answers

Answer:

61=x cortical opposite angle

61=y corresponding angle

Answer:

61=x and 61=y

Step-by-step explanation:

I think that bout right

Use the distributive property to write an equivalent expression. 2.5 ( w + 3 )

Answers

Answer:

2.5w + 7.5

Step-by-step explanation:

2.5(w+3)

2.5w+7.5

No need to show work just need answer.

Answers

Can I have brainliest since u did t put a picture :)

Graph the line that passes through the points (-2, - 7) and (5,0) and determine
the equation of the line.

Answers

Equation of the line is: y = x - 5

Work shown in pic:

The measure of the exterior angle of the triangle is

Answers

Answer:

Step-by-step explanation:

Hi! So far, in the question provided, you have two of the three angles that you need to find x. To find x, you need all three interior angles of the triangle. When you look at the image, you can see that (7x-16) is a complementary angle to the interior angle we need. To find the final angle of the triangle, you are going to take 180 - (7x-16). This is going to give you the interior angle to be (196-7x).

Now that we know all three angles for the inside of the triangle, we can find the value of x. We know that all of the angles inside of a triangle add up to 180 degrees. This is going to help you find x:

180 = (x+8) + (4x) + (196-7x)

180 = x + 8 + 4x + 196 - 7x

180 = -2x + 204

-24 = -2x

12 = x

Now that we have have the value of x, we can solve for the exterior angle.

(7(12) - 16) = 68 degrees

Hope this helps!

Answer:

68°

Step-by-step explanation:

The sum of two interior angles in a triangle is equal to an exterior angle that's supplementary to the third interior angle

4x + x + 8 = 7x - 16 add like terms

5x + 8 = 7x - 16 export like terms to the same side

8 + 16 = 7x - 5x

24 = 2x divide both sides by 2

12 = x

The measure of exterior angle 7x - 16 ➡ 7 × 12 - 16 = 68°

are the 2 triangles congruent? ​

Answers

Answer:

Yes

Step-by-step explanation:

They are just in the opposite side

10. Four bagels and eight muffins cost $16. Six bagels
and five muffins cost $13.50 How much does a bagel
cost? How much does a muffin cost?

Answers

bagel=$1
muffin=$1.50

Need help resolving the following quadratic expressions:

a) 4x -4x -15 = 0

b) 2y +7y + 3 +0

Answers

Answer:

first answer for (a) is -15=0

second answer for (b) is =9y+3

Step-by-step explanation:

its juss so easy

what is the variable to 3x-5 ​

Answers

Answer:

3 (2x + 3) + 11 = –2 (2x - 6) - 18

Answer:

-15

Step-by-step explanation:

this is because when a negative multiplies with a positive ....it gives an answer of negative

x
1
2
3
4
5
y
13
16.9
21.97
28.56
37.13
Graph the function shown by the table. Is the function linear or nonlinear?

Yes
NO

Answers

PLEASE HELP ME!!!!!!!!!!!!!!$!!!!!!!!

Can somebody please help me

Answers

Answer:

Step-by-step explanation:

the shaded area would be

200*75=15,000 ft^2

from this are we have to subtract the are of the house and of the  garage

house= 40*40=1600

garage=24*30=720

house and garage= 1600+720=2320

15,000-2320=12,680

Answer:

12680 ft²

Step-by-step explanation:

First we will find the area of the house and then the garage.

They're both rectangles so we should use this formula [tex]A=l*w[/tex].

Area Of The House:

[tex]A=l*w\\A=40*40\\A=1600ft^2[/tex]

Area Of The Garage:

[tex]A=l*w\\A=24*30\\A=720ft^2[/tex]

Adding the two areas:

Now we have to add the two together to find how much of the space they take up in the yard.

[tex]1600+720=2320[/tex]

Area Of The Yard:

[tex]A=l*w\\A= 200*75\\A=15000ft^2[/tex]

Subtracting The Two Areas To The Yard's Area:

[tex]15000-2320=12680ft^2[/tex] and that is it!

An item sells for $30 and is on sale for 15% off and the sales tax is 8.2%. What is the final cost to the nearest cent? (Calculate the discounted cost first and then find the tax on the discounted cost.)

Answers

Answer:

$27.59

Step-by-step explanation:

You multiply 30 dollars by 0.85 (1-0.15, with the 0.15 being for the 15 percent off). Which gets you to $25.50, and you then multiply it by 1.082 (the 1.082 being the tax, and the .082 part equating to the 8.2% tax percentage).

Other Questions
BRANDI WANTS TO CALCULATE THE AVERAGE SPEEDOF A CAR THAT TRAVELED 275 MILES IN THREEHOURS. WHICH FORMULA BELOW IS CORRECT?A. S = 3 x 275B. S = 3/275C. S = 275/3D. S = 3-275MISTER SCIENCE What fraction of the students finished in less than 20 minutes Which words could replace the word leverishly, in the sentence? Can somebody just give me 2 sentences on why Christmas trees are the best lol? Each batch of cookies is 8.9 oz. What is the weight of 2.5 batches of cookies? *help me with this please and thank you* in 2017, the price of a Forever stamp from the United States Postal Service was $0.49. How many Forever stamps could be purchased for $30.38? Tell me about a time when you made a mistake.. How did you find it and what did you do to correct it? HEY CAN ANYONE PLS ANSWER DIS SPANISH WORK!!!!! What is the primary difference between electronic medical records (EMRS) and electronic health records (EHRs)?A. EMRs are covered under HIPAA, but EHRs have more privacy restrictions,B. EMRs are comprehensive, but EHRs are specific to a single event or diagnosis.C. EMRs are given to the patient, but EHRs are held only by medical professionals.D. EMRs are used within a practice, but EHRs can be shared among medical practices. A clothing store used the sign shown below to advertise a discount on shirts. DISCOUNT Buy Two ShirtsGet 50% Off Third ShirtCelia wants to buy three shirts, which were originally priced $56 each. The store will discount theprice of the third shirt and then apply a 7% tax to the total cost of all three shirts. Including the tax.what will be the mean cost of each shirt? (Round your answer to the nearest cent).Including the tax, the mean cost of each shirt is $___? Three times Jamie's age plus 3 is Maria's age. Maria is 48. What is Jamie's age? 11/30 long division as a decimal Heridan Company is considering the following alternatives: Alternative AAlternative B Revenues$64000$72000 Variable costs 38400 38400 Fixed costs 10000 16000 What is the incremental profit? Which of the following best describes hair?Hair is made of mostly tightly packed living cells.Hair does not help control body temperature.Hair growth occurs in the hair follicle.Hair grows at 2.5mm a month.Mark this and returnSave and ExitNextSubmit please Quick Someone help me correct these sentences1. an healthy bone creats new cells all the time (4 errors)2. he handed a carton of milk to i cause strong bones need calcium phosphorus and other minerals (6 errors) HURRY PLZ It is important to practice before a multimedia presentation in order toO identify sections that need to be revised.O memorize the presentation in its entirety.O determine the most appropriate audienceO decide the purpose of the presentation. need rn! please help, thx :) What is the slope of the graph of 4x + 3y = 15? Complete the following sentence.Some manufactured dams produce BLANK power in addition to retaining water. Consider an asset with a beta of 1.2, a risk-free rate of 4.4%, and a market return of 12.4%. What is the reward to risk ratio?